Merge remote branch 'public/master'
[course.git] / latex / problems / Serway_and_Jewett_4 / problem20.21.tex
1 \begin{problem*}{20.21}
2 The potential in a region between $x=0$ and $x=6.00\U{m}$ is $V =
3 a+bx$, where $a = 10.0\U{V}$ and $b = -7.00\U{V/m}$.  Determine
4  \Part{a} the potential at $x = 0$, $3.00\U{m}$, and $6.00\U{m}$; and 
5
6  \Part{b} the magnitude and direction of the electric field at $x =
7 0$, $3.00\U{m}$, and $6.00\U{m}$.
8 \end{problem5} % problem 20.21
9
10 \begin{solution}
11 \Part{a}
12 Simply plugging into their $V(x)$ formula
13 \begin{align}
14  V(0\U{m}) &= \ans{10.0\U{V}} \\
15  V(3.00\U{m}) &= 10.0\U{V} - 21.0\U{V} = \ans{-11\U{V}} \\
16  V(6.00\U{m}) &= 10.0\U{V} - 42.0\U{V} = \ans{-32\U{V}}
17 \end{align}
18
19 \Part{b}
20 Using $E_x = -dV/dx$ we have
21 \begin{equation}
22  E = - \frac{d}{dx}(a+bx) = -b = \ans{7.00\U{V/m}}
23 \end{equation}
24 At any point for $0 \le x \le 6.00\U{m}$.
25 \end{solution}